Second degrée et dérivation


  • L

    j'ai un autre exercice dont je n'arrive pas du tout si vos pouviez m'aidez SANS ME DONNER LES REPONSES j'aimerais comprendre et ensuite essayer de trouver par moi meme mais si j'y arrive vraiment pas bah y'a plu qu'a me donner la réponse et jessayerai de comprendre merki d'avance !!!

    Une petite entreprise fabrique et vend des chaises.
    Le cout de fabrication en euros de q chaises est donnée par :
    C(q) = 0,1q² + 4q + 1000
    Chaque chaises est vendue 29 euros
    1- Pour quels nombres de chaises fabriquées le cout est il inférieur à 2000 euros ?
    2- a) Exprimer le bénéfice B(q) en fonction de q
    b) Pour quels nombres de chaises fabriquées et vendues le bénéfice est il positif ou nul ?
    3- Etudier le sens de variation de la fonction B. En déduire le bénéfice maximal

    Pour toute info moi je sais juste que pour calculer le bénéfice la formule c'est :
    B(q) = R(q) - C(q)

    Merci de m'aider au plus vite


  • J

    Salut.

    Le coût de fabrication: C(q)=0,1q²+4q+1000
    Nombre de chaises fabriquées: q

    On veut que le coût soit inférieur à 2000€. Ca t'amène à poser une inéquation non?

    a) Qu'est-ce que le bénéfice? C'est l'argent qu'il te reste après avoir vendu les chaises: donc le bénéfice c'est ce que tu gagnes en vendant les chaises, moins toutes les dépenses annexes ici(le coût de fabrication dans cet exercice).

    b) Une petite inéquation?

    1. Là il te faut l'expression de B, donc je ne dis rien.

    @+


  • L

    pour lexo g trouvé comme solution S -57 ; 17
    s ke c exact por le 1 ?


  • J

    Salut.

    Ton autre question est déplacée ici: http://www.math...start-0.html.

    Pour la 1), pour q=0 par exemple, le coût est de 1000€. Ce qui est bien inférieur à 2000€. Donc je ne suis pas d'accord. Prends la peine de bien écrire tes résultats s'il-te-plaît, ainsi que l'inéquation à résoudre.

    @+


  • L

    moi j'ai fait 0,1q²+4q+1000 < 2000
    donc je me sui trompé c <2 ?
    c sa ?


  • J

    Salut.

    0,1q²+4q+1000≤2000 (quand on dit "inférieur" sans le "strictement", il faut comprende inférieur ou égal). Sinon je suis d'accord avec l'inéquation.

    Je me répète, écrit correctement ton résultat. Et sans langage SMS.

    Si tu me dis que c'est inférieur à 2, ben moi je prends q=-30000000000 et je pense que dans ce cas le coût est supérieur à 2000€.

    @+


  • L

    j'ai fait :

    0,1q² + 4q -1000 <= 2000
    equiv/ 0.1q² + 4q -1000
    donc on cacul (delta) = b²-4ac ce qui donne 56
    donc comme (delata) est supérieur à 0 donc on a 2 solution x1 et x2
    et je trouve donc -57 pour x1 et our x2 = 17


  • J

    Salut.

    0,1q²+4q+1000≤2000 equiv/ 0.1q²+4q-1000≤0

    Ensuite, tu as dû te tromper dans le calcul du discriminant de 0.1q²+4q-1000.

    Pour finir, tu cherches q tel que 0.1q²+4q-1000≤0 et non tel que 0.1q²+4q-1000=0. C'est bien une inégalité.

    Détaille ton calcul, sinon on va tourner en rond. Là je ne peux pas te dire où tu t'es trompé. Enfin... si. Là je devine. -4ac n'est pas égal à 40, mais à 400.

    @+


  • L

    0,1q² + 4q + 1000 <= 2000 (esce qu'on le garde le "strictement")
    0,1q² + 4q - 1000
    donc ensuite je calcul (delta) qui est égale à b² -4ac
    donc = 4² - 40,1(-100) donc = 56
    delta est donc supérieur à 0
    on a donc 2 solutions :
    x1 = -b- sqrtsqrtsqrt(delta) / 2a = -4- sqrtsqrtsqrt56) / 20,1 = -57
    x2 = -b+ sqrtsqrtsqrt(delta) / 2a = -4+ sqrtsqrtsqrt56) / 2
    0,1 = 17
    donc je trouve ces deux solutions


  • J

    Salut.

    Lis ce que j'ai écrit plus haut. Tout est marqué. Et c=-1000 et non -100.

    @+


  • L

    [je ne comprend plus rien je suis totalement perdu désolé


  • J

    Salut.

    En ce qui concerne l'inéquation, j'ai écrit:

    0,1q²+4q+1000≤2000 equiv/ 0,1q²+4q-1000≤0

    Ce qui répond à ta question d'inégalité stricte ou non.

    Tu calcules le discriminant de 0.1q²+4q-1000 n'est-ce pas?

    Δ=b²-4ac, avec:

    b=4
    a=0,1=1/10
    c=-1000

    Donc Δ=4²-40,1(-1000). Fini le calcul.

    @+


  • L

    Jeet-chris
    Salut.

    En ce qui concerne l'inéquation, j'ai écrit:

    0,1q²+4q+1000≤2000 equiv/ 0,1q²+4q-1000≤0

    Ce qui répond à ta question d'inégalité stricte ou non.

    Tu calcules le discriminant de 0.1q²+4q-1000 n'est-ce pas?

    Δ=b²-4ac, avec:

    b=4
    a=0,1=1/10
    c=-1000

    Donc Δ=4²-40,1(-1000). Fini le calcul.

    @+

    Ce qui donne 56
    c sa ?


  • L

    les 2 soluton ne servent donc à rien ?


  • J

    Salut.

    Je t'ai dit que tu t'es trompé. Ne te borne pas à dire que tu as raison.

    Δ=4²-40,1(-1000)=16+400=416≠56

    Quand je te dis de refaire le calcul, refait-le au lieu de me balancer les mêmes valeurs.

    Je n'ai pas dit que les racines sont inutiles, mais que les tiennent ne sont pas bonnes.

    @+


  • L

    Jeet-chris
    Salut.

    Je t'ai dit que tu t'es trompé. Ne te borne pas à dire que tu as raison.

    Δ=4²-40,1(-1000)=16+400=416≠56

    Quand je te dis de refaire le calcul, refait-le au lieu de me balancer les mêmes valeurs.

    Je n'ai pas dit que les racines sont inutiles, mais que les tiennent ne sont pas bonnes.

    @+

    donc x1 = -b- sqrtsqrtsqrt(delta)/2a = -4- sqrtsqrtsqrt416) / 20,1 = 122 (121,980)
    x2 = -b+ sqrtsqrtsqrt(delta)/2a = -4+ sqrtsqrtsqrt416)/2
    0,1 = 82 (81,980)

    s ke c correct maintzeant ?


  • J

    Salut.

    Ce sont les bonnes racines, mis à part que tu as oublié le signe négatif dans a valeur de x1x_1x1: c'est ≈-121,98. Une faute d'inattention en somme.

    Maintenant, il faut répondre à la question. Il faut que tu donnes toutes les valeurs de q telles que le coût soit inférieur à 2000€.

    Comme on crée un nombre positif de chaises en général 😁 , il faut prendre tous les q de 0 à ... ? Attention à ta réponse! Sinon tu vas te faire avoir à cause de ta valeur approchée.

    @+


  • L

    Jeet-chris
    Salut.

    Ce sont les bonnes racines, mis à part que tu as oublié le signe négatif dans a valeur de x1x_1x1: c'est ≈-121,98. Une faute d'inattention en somme.

    Maintenant, il faut répondre à la question. Il faut que tu donnes toutes les valeurs de q telles que le coût soit inférieur à 2000€.

    Comme on crée un nombre positif de chaises en général 😁 , il faut prendre tous les q de 0 à ... ? Attention à ta réponse! Sinon tu vas te faire avoir à cause de ta valeur approchée.

    @+

    bah entre 0 et 81


  • L

    R(q) = 29 représente le prix de vente de q chaises
    et B(q) = R(q) - C(q)
    donc je calcul c sa la suite

    au fait la 1 c bon ou pas fo ke je dise kwa cmme phrase ?


  • J

    Salut.

    C'est bien de 0 à 81.

    En revanche, R(q) n'est pas bien exprimé.

    @+


  • L

    donc je l'exprime en fonctionde q pas de 29


  • L

    B(q) = R(q) - C(q)
    = q - 0,1q² + 4q + 1000
    = -0,1q² + 5q + 1000
    c correct ?


  • L

    non me suis trompé g trouvé pour la 2 :
    a) B(q) = -0,1q² + 25q -1000
    b) j'ai calculer delta donc = 225
    puis x1 = 200
    x2 = 50

    pour la 3 :
    je ne sauis pas si c sa
    g calculez le sommet donc alpha et j'ai trouvé 125 puis g fai un tableau de vaiation mais bon je ne toruve pas !!!!


Se connecter pour répondre